Jump to content
BrainDen.com - Brain Teasers
  • 0


unreality
 Share

Question

Today on my first day of break I found myself bored already and messing around with exponential equations.

A lot of very simple equations, such as a^b = b^a, are not solvable by typical algebra, although sometimes variants are. For example, it's possible if b is a known multiple of a... ie, x^(kx) = (kx)^x, solving for x in terms of k, you get x = k^(1/(k-1)). But most equations with exponentials, including things as simple as e^x = x (which has no real solution and only one complex solution, x = 0.31813150520476413531 - 1.3372357014306894*i) are just impossible with standard algebra unless you're clever or have a computer.

The point here is to not google/wolframalpha/maple/matlab these but instead try to figure them out yourselves, without using the kinds of non-elementary functions people have devised (ie, LambertW). For an example of what I mean, see the hint I posted for #1.

Roughly in order of difficulty...

[1]

e^y = x*y

Find y in terms of x, ie, find the function y(x)

All you need is one function

[2]

For a function f(x) = x^(x^(x^(x^(x^(...

which goes on to infinity,

find the derivative f ' (x)

And also find the values of x such that f(x) is finite.

[3]

Now introducing the "W" function, which solves this equation: z = W*e^W. So W(0) is the solution for W of 0 = W*e^W, i.e. the value is W(0) = 0. Other values are W(-pi/2) = i*pi/2, and W(-1/e) = -1, and W(1) is the omega constant (.56714).

Armed with this "non elementary" function, solve this analog of the quadratic equation for x:

a*ln(ln(x)) + b*ln(x) + c*x = 0

(hint: try it with blnx + cx = 0 first)

I haven't gotten this one yet :lol: :lol: :lol:

[4]

Use W to invert these functions:

f(x) = x^x

f(x) = x^(x^(x^(x^(x^(x^(x^(x^(x^(x^(x^(x^(x^(x^(x^(x^(x^(x^(...

f(x) = x*ln(x)

Link to comment
Share on other sites

16 answers to this question

Recommended Posts

  • 0

I'm not sure if I understand this correctly but it seems like f(x) = lim as n-> infinity of the sequence of functions defined by f1= x, fn+1= x^(fn). If that is the case then f(x) is undefined for x<0, f(x)=0 for x=0, f(x) = 1 for 0<x<=1, and f(x) diverges to positive infinity for x>1. This would make f'(x)=1 where it exists. But this seems a lot simpler than problem 1 so it's probably wrong to some extent.

Link to comment
Share on other sites

  • 0

I'm not sure if I understand this correctly but it seems like f(x) = lim as n-> infinity of the sequence of functions defined by f1= x, fn+1= x^(fn). If that is the case then f(x) is undefined for x<0, f(x)=0 for x=0, f(x) = 1 for 0<x<=1, and f(x) diverges to positive infinity for x>1. This would make f'(x)=1 where it exists. But this seems a lot simpler than problem 1 so it's probably wrong to some extent.

yeah it's a bit more complex than that :)

Link to comment
Share on other sites

  • 0

I came up with f'(x) = 1 / x^(f(x)-1)

The function f(x) is finite for all 0 <= x <= sqrt(2) and infinite for all x>sqrt(2)

sqrt(2) is 1.41421356

However,

1.44466786

And regarding the derivative, that may be another valid way of expressing it, I don't know without your steps, but if you:

you might get something like

f'(x) = f(x)^2 / (x - x*f(x)*ln(x))

Link to comment
Share on other sites

  • 0

e^y = x*y

=> y = log (x*y)

=> y = log x + log y

=> y = log x + log (log x + log y)

=> y = log x + log (log x + log (log x + log (log x + log (log x + ....

now, if x = e^z then

y = z + log (z + log (z + log ( z + ...

Link to comment
Share on other sites

  • 0

sqrt(2) is 1.41421356

However,

1.44466786

I stand corrected.

... the answer is sqrt(e)

And regarding the derivative, that may be another valid way of expressing it, I don't know without your steps, but if you:

you might get something like

f'(x) = f(x)^2 / (x - x*f(x)*ln(x))

No, mine is definitely wrong. I haven't done any advanced calculus in a long time, so I tried to retrace my own steps and couldn't repeat it. Found a mistake. However, I couldn't get your answer either. Care to post how did you get your answer?

Link to comment
Share on other sites

  • 0

since it's powers, so any value > 1 should reach to countable infinity with sufficiently large iterations...

That's what intuition may tell you, but not true. Try it with sqrt(2).

sqrt(2) = 1.414214

sqrt(2)^sqrt(2) = 1.632527

sqrt(2)^ 1.632527 = 1.76084

sqrt(2)^1.76084 = 1.840911

keep going and it converges on 2.

However sqrt(2) isn't quite the limit, you can go higher yet. (But not so high as sqrt(e) which was k-man's last guess)

I stand corrected.

... the answer is sqrt(e)

sqrt(e) = e^(1/2) is too high, it doesn't work. You're getting warmer though :)

No, mine is definitely wrong. I haven't done any advanced calculus in a long time, so I tried to retrace my own steps and couldn't repeat it. Found a mistake. However, I couldn't get your answer either. Care to post how did you get your answer?

y = x^(x^(x^(...

ln(y) = x^(x^(x..)))))..) * ln(x) = y * ln(x)

so we have

ln(y) = y*ln(x)

now take the derivative with respect to x:

(dy/dx) / y = (dy/dx)*ln(x) + y/x

from here it's just algebra:

-y/x = (dy/dx) * (ln(x) - 1/y)

(dy/dx) = -y / ( x*(ln(x) - 1/y) )

f ' (x) = -f(x) / (xlnx - x/f(x))

which is the same as:

f ' (x) = f(x)^2 / (x - x*ln(x)*f(x) )

Link to comment
Share on other sites

  • 0

That's what intuition may tell you, but not true. Try it with sqrt(2).

sqrt(2) = 1.414214

sqrt(2)^sqrt(2) = 1.632527

sqrt(2)^ 1.632527 = 1.76084

sqrt(2)^1.76084 = 1.840911

keep going and it converges on 2.

However sqrt(2) isn't quite the limit, you can go higher yet. (But not so high as sqrt(e) which was k-man's last guess)

hmm, in that case...

y = x ^ (x ^ (x ^ ( x ^ ...

=> y = x ^ y

=> x = y ^ 1/y

at y = (inf), you are asking me to define 1/(inf) and (inf) ^ (1/(inf))

No problem, let me try...

Link to comment
Share on other sites

  • 0

However sqrt(2) isn't quite the limit, you can go higher yet. (But not so high as sqrt(e) which was k-man's last guess)

sqrt(e) = e^(1/2) is too high, it doesn't work. You're getting warmer though :)

I saw that the largest finite number that this sequence converges to is e. So I just wrote sqrt(e) since I saw that sqrt(2) converges to 2. The correct answer (and I verified it) is e1/e:duh:

Link to comment
Share on other sites

  • 0

yep good job :) There hasn't been much progress so I'll go ahead and post the answers

[1]

e^y = x*y

Find y in terms of x, ie, find the function y(x)

y = log x + log (log x + log (log x + log (log x + log (log x + ....

KlueMaster got it!

[2]

For a function f(x) = x^(x^(x^(x^(x^(...

which goes on to infinity,

find the derivative f ' (x)

And also find the values of x such that f(x) is finite.

f'(x) = f(x)^2 / (x - x*f(x)*ln(x))

the bounds of convergence are e^(-e) and e^(1/e)

[3]

Now introducing the "W" function, which solves this equation: z = W*e^W. So W(0) is the solution for W of 0 = W*e^W, i.e. the value is W(0) = 0. Other values are W(-pi/2) = i*pi/2, and W(-1/e) = -1, and W(1) is the omega constant (.56714).

Armed with this "non elementary" function, solve this:

(hint: try it with blnx + cx = 0 first)

x = (b/c) * W(c/b)

[4]

Use W to invert these functions:

f(x) = x^x

f(x) = x^(x^(x^(x^(x^(x^(x^(x^(x^(x^(x^(x^(x^(x^(x^(x^(x^(x^(...

f(x) = x*ln(x)

If g(x) is the inverse function of f(x) then:

f(x) = x^x

g(x) = ln(x) / W(ln(x))

f(x) = x^(x^(x^(x^(x^(x^(x^(x^(x^(x^(x^(x^(x^(x^(x^(x^(x^(x^(...

You don't even need the W function for this one:

g(x) = e^( ln(x) / x) = x^(1/x)

f(x) = x*ln(x)

g(x) = x / W(x)

Link to comment
Share on other sites

  • 0

That's what intuition may tell you, but not true. Try it with sqrt(2).

sqrt(2) = 1.414214

sqrt(2)^sqrt(2) = 1.632527

sqrt(2)^ 1.632527 = 1.76084

sqrt(2)^1.76084 = 1.840911

keep going and it converges on 2.

However sqrt(2) isn't quite the limit, you can go higher yet. (But not so high as sqrt(e) which was k-man's last guess)

sqrt(e) = e^(1/2) is too high, it doesn't work. You're getting warmer though :)

y = x^(x^(x^(...

ln(y) = x^(x^(x..)))))..) * ln(x) = y * ln(x)

so we have

ln(y) = y*ln(x)

now take the derivative with respect to x:

(dy/dx) / y = (dy/dx)*ln(x) + y/x

from here it's just algebra:

-y/x = (dy/dx) * (ln(x) - 1/y)

(dy/dx) = -y / ( x*(ln(x) - 1/y) )

f ' (x) = -f(x) / (xlnx - x/f(x))

which is the same as:

f ' (x) = f(x)^2 / (x - x*ln(x)*f(x) )

ya i got f'(x) by doing it implicitly but like k-man, my intuition tells me sqrt(2) might work and tend to be finite, are u sure about d answer?

Edited by dark_magician_92
Link to comment
Share on other sites

  • 0

ya i got f'(x) by doing it implicitly but like k-man, my intuition tells me sqrt(2) might work and tend to be finite, are u sure about d answer?

One way to show the e^(1/e) limit is this...

so we have y' = (y^2) / (x - x y ln x)

we'll have critical points where y' is 0 or undefined.

0 when y^2 = 0, so y = 0. We find the inverse function x = y^(1/y) and y=0, so 0^(1/0) isn't really defined. So look just at the bottom:

x(1 - ylnx) = 0

x = 0 and 1 = ylnx

1 = ylnx

raise e to both sides to get:

e = y

then use the inverse (x = y^(1/y)) to get x = e^(1/e)

That's one way to do it, kind of sloppy though

Link to comment
Share on other sites

  • 0

One way to show the e^(1/e) limit is this...

so we have y' = (y^2) / (x - x y ln x)

we'll have critical points where y' is 0 or undefined.

0 when y^2 = 0, so y = 0. We find the inverse function x = y^(1/y) and y=0, so 0^(1/0) isn't really defined. So look just at the bottom:

x(1 - ylnx) = 0

x = 0 and 1 = ylnx

1 = ylnx

raise e to both sides to get:

e = y

then use the inverse (x = y^(1/y)) to get x = e^(1/e)

That's one way to do it, kind of sloppy though

thanks a lot, i solved the problem upto that "ylnx=1", but couldnt solve further. Thanks n I feel happy now to have joined brainden :)

Link to comment
Share on other sites

Join the conversation

You can post now and register later. If you have an account, sign in now to post with your account.

Guest
Answer this question...

×   Pasted as rich text.   Paste as plain text instead

  Only 75 emoji are allowed.

×   Your link has been automatically embedded.   Display as a link instead

×   Your previous content has been restored.   Clear editor

×   You cannot paste images directly. Upload or insert images from URL.

Loading...
 Share

  • Recently Browsing   0 members

    • No registered users viewing this page.
×
×
  • Create New...